LSAT and Law School Admissions Forum

Get expert LSAT preparation and law school admissions advice from PowerScore Test Preparation.

 taylorballou
  • Posts: 18
  • Joined: Feb 18, 2017
|
#40002
Hello,

Could someone explain the answer to this question? I completed this section as a timed exercise and skipped this question as I was having too difficult of a time figuring it out. Initially, I thought the answer choice was E, but decided this was probably not correct because the stimulus mentions "development costs for new vaccines," not "the costs of administering a vaccine."

Is C correct because the author claims medicines are more profitable than vaccines because they are administered more times, and if pharmaceutical companies market products that are not medicines nor vaccines, they have sold products that have the potential of not being administered at all?

Thanks,

Taylor
 AthenaDalton
PowerScore Staff
  • PowerScore Staff
  • Posts: 296
  • Joined: May 02, 2017
|
#40026
Hi Taylor,

Answer choice (A) is actually the correct answer to this one! The pharmaceutical company rep argues that their development of vaccines should be subsidized, since each vaccine will only be administered once person (in contrast to a medicine that has to be taken daily to manage a chronic illness). The pharma rep concludes that sales will be lower since each person only needs to receive a vaccine once per lifetime.

Since the argument boils down to volume, or the number of doses that can be sold of a particular medicine, any information that indicates vaccines will be a high-volume medicine will weaken the argument.

Answer choice (A) weakens the pharma rep's argument by pointing out that vaccines will be administered to many, many more people than any other drug. Consider this example:

Rexx Pharma has developed two drugs, the Polio Vaccine (which must only be administered once in a person's lifetime) and a pill to treat Disease A (which must be taken daily). But then consider that perhaps 99 percent of the population receives the Polio Vaccine, while only a fraction of a percent of the population will ever develop Disease A. Even though the vaccine only needs to be administered once, if a huge swath of the population receives the vaccine, the number of injections sold will exceed the number of doses purchased for Disease A.

Answer choice (C) doesn't help us weaken the argument. The pharma rep is concerned with the return-on-investment for vaccines, saying that the company will have to spend a huge amount of money developing the vaccines, which customers will only purchase one time. He is using the example of medicines for chronic illnesses just as a point of comparison, where one customer will purchase many doses over their lifetime. The fact that a pharma company also markets other types of products doesn't impact the pharma rep's argument that his company's return-on-investment for vaccines will be low.

I hope that helps clarify things. Good luck studying!
 taylorballou
  • Posts: 18
  • Joined: Feb 18, 2017
|
#40187
Oh oops! My mistake. The explanation definitely helped then haha. Thank you!
 Tony_Stark
  • Posts: 15
  • Joined: Jun 29, 2017
|
#41209
I have a quick question about A.

In the premise, the director says "the marketing of vaccines promised to be less profitable than the marketing of any other pharma..."

The answer A) almost says that it will be more profitable.

I'm wondering why this is not directly contradicting the premise?

Thanks in advance.
 Eric Ockert
PowerScore Staff
  • PowerScore Staff
  • Posts: 164
  • Joined: Sep 28, 2011
|
#41762
Hi Tony!

The statement you are referring to is technically an intermediate conclusion of the argument. The flow of the argument is essentially:

Prem: Vaccines are only administered to patients once
Prem: Other medicines are administered to patients multiple times.
Int. Conclusion (1): Therefore, sales of vaccines are likely to be lower than other products.
Int. Conclusion (2): Therefore, marketing of vaccines will be less profitable than other products.
Conclusion: Development costs for vaccines should be subsidized by the government.

Answer choice (A) really targets the first intermediate conclusion above, that sales are likely to be lower. Even though vaccines are only given to each patient once, if answer (A) is true then vaccine sales don't appear to necessarily be lower than for other products (although they still could be). This then questions whether they are less profitable (but doesn't necessarily "directly contradict" it) which questions the main conclusion that development costs should be subsidized.

Hope that helps!
 vbkehs
  • Posts: 31
  • Joined: Mar 31, 2020
|
#76099
Based on the explanation below, my understanding is that this answer choice is true because it weakens it the most out of the other choices -- not 100%, but still greater than nothing. Is that correct?

I'm having trouble finding where I went wrong in my thinking. For example, my spouse has a chronic disease that requires lifelong treatment. I dismissed answer choice A because I'm aware of the possibility that a pharma company can distribute, for example, 100 vaccines to 100 people. However, they could also administer drugs for chronic illnesses to 5 people at a rate of 2x/year -- the amount of doses which, over the collective course of a lifetime, would exceed 100 vaccines.

I chose E by elimination because of my reasoning above. Does E weaken the argument at all, and does so less than A, or is it irrelevant? Thanks in advance!
Eric Ockert wrote:Hi Tony!

The statement you are referring to is technically an intermediate conclusion of the argument. The flow of the argument is essentially:

Prem: Vaccines are only administered to patients once
Prem: Other medicines are administered to patients multiple times.
Int. Conclusion (1): Therefore, sales of vaccines are likely to be lower than other products.
Int. Conclusion (2): Therefore, marketing of vaccines will be less profitable than other products.
Conclusion: Development costs for vaccines should be subsidized by the government.

Answer choice (A) really targets the first intermediate conclusion above, that sales are likely to be lower. Even though vaccines are only given to each patient once, if answer (A) is true then vaccine sales don't appear to necessarily be lower than for other products (although they still could be). This then questions whether they are less profitable (but doesn't necessarily "directly contradict" it) which questions the main conclusion that development costs should be subsidized.

Hope that helps!
 Adam Tyson
PowerScore Staff
  • PowerScore Staff
  • Posts: 5153
  • Joined: Apr 14, 2011
|
#76882
I don't see E as having any effect on the argument, vbkehs, because the issue is the profit from sales of a medication or vaccine, not from the administration of that medication or vaccine. Also, keep in mind that a good Weaken answer doesn't need to disprove a conclusion or completely destroy the reasoning in the argument, but only, as Eric put it, question the strength of the argument. It needs to raise some doubts about the reasoning that led to the conclusion.

Also, one more thing, and that is that we aren't trying to determine whether a Weaken answer is true. We are supposed to start with the idea that ALL of the answers are true. The question you have to ask yourself is whether that answer, taken as true, hurts the argument. Answer A does that, by undermining the claim about the sales numbers for a vaccine vs a medication for a disease or chronic illness. The other answers, even if they are true, do not raise any doubts about this argument, because they all address issues that are not relevant to the argument.
User avatar
 JosephRusciano
  • Posts: 1
  • Joined: Dec 08, 2021
|
#92495
What is the assumption in this question? Is it that the initial price of the vaccine will be lower as well?
User avatar
 Dave Killoran
PowerScore Staff
  • PowerScore Staff
  • Posts: 5853
  • Joined: Mar 25, 2011
|
#92502
Hi Joseph,

No, they don't make that assumption actually! Instead, they assume the target vaccine user group is of some similar size to the group that uses "medicines that combat diseases and chronic illnesses." Answer (A) then blows up that idea by showing that the user groups are markedly different.

Thanks!

Get the most out of your LSAT Prep Plus subscription.

Analyze and track your performance with our Testing and Analytics Package.